LSAT and Law School Admissions Forum

Get expert LSAT preparation and law school admissions advice from PowerScore Test Preparation.

 Administrator
PowerScore Staff
  • PowerScore Staff
  • Posts: 8948
  • Joined: Feb 02, 2011
|
#23056
Complete Question Explanation

Assumption. The correct answer choice is (C)

The stimulus explains that when lava solidifies, it becomes magnetized in the direction of Earth's magnetic field. The argument observes that lava flows of very different ages have magnetized in significantly different directions, but flows of close ages often have very magnetized in similar directions. On that basis, the stimulus concludes that the change in Earth's magnetic field takes place gradually.

The argument probably makes a causal assumption. Since the Earth's magnetic field causes the direction of magnetization upon lava solidification, the argument supposes that all the gradually changing data on old flows reflects that cause. You are asked to identify that necessary assumption.

Answer choice (A): The argument was that lava indicates something about Earth's magnetic field, not that lava is the only or best way to measure the Earth's magnetic field. As long as lava is sufficient, it does not matter whether other methods would work, so the argument does not need to assume that only lava would work.

Answer choice (B): As delivered in this choice, the only legitimate interpretation of "consistency" has to do with thickness and composition. The use of "consistency" in this choice should not be interpreted as having anything to do with the direction of magnetization.

Answer choice (C): This is the correct answer choice. Since scientists take the readings to indicate a very distant cause, scientists have to assume that nothing unpredictable happened in the meantime that would make it virtually impossible to be certain that the readings reflect the distant cause. If something unpredictable did occur in the meantime, the readings might not indicate anything certain about the original state of the lava flow.

Answer choice (D): The argument does not need to assume anything about the frequency of eruptions. As long as some lava exists, the measurements might be possible.

Answer choice (E): The argument did not suggest that magnetized rocks serve to magnetize the lava; the argument clearly implies that Earth's magnetic field causes the magnetization. This choice is wrong, because it weakens the idea that the readings necessarily indicate the direction of Earth's magnetic field. Even if you incorrectly argue that the magnetized rocks would align with the magnetic field, there is no reason to believe that those rocks are necessary.
 moshei24
  • Posts: 465
  • Joined: Mar 20, 2012
|
#4536
Can someone explain the answer to question 40 on page 5-63 in lesson five's homework?

The explanation says that the scientists would need to assume that nothing unpredictable happened; yet, in the correct answer choice is says that "not all solidified lava...changed...unpredictably." Doesn't "not all" mean that some did, which would mean that something unpredictable happened? I see that it's the best answer, but it still seems to be faulty.

Thanks.
 Steve Stein
PowerScore Staff
  • PowerScore Staff
  • Posts: 1153
  • Joined: Apr 11, 2011
|
#4539
In that example, the author points out that when lava becomes solid, it gets magnetized in the same direction as the Earth's magnetic field.

The solidified lava from different volcanoes' magnetically points in different directions, so the author concludes that the Earth has been changing its magnetic field (causing all those different directions).

The question asks for a necessary assumption, so we can apply the Assumption Negation technique: the correct answer choice will provide an assumption which, when negated, will weaken the author's conclusion. Correct answer choice C, when negated, would provide the following:

All solidified lava has changed its magnetic direction unpredictably.

If, through time, all solidified lava went through random changes in magnetic direction, then the different magnetic directions would no longer necessarily reflect the direction of the earth at the time of each respective volcano.

Tough question! Let me know whether it's clear--thanks!

~Steve
 moshei24
  • Posts: 465
  • Joined: Mar 20, 2012
|
#4575
Sorry for the late response. I thought I would get an email when my post got a response, but did not.

I see what you're saying. That is what I understood, as well. However, I still don't see how that answer isn't fault. If the assumption is that "not all," which means "some," lava has changed direction of its magnetization unpredictably then that would weaken the argument. The possibility that some lava changed direction unpredictably would bring an issue to the argument that they don't know if the change was due to the Earth shifting or the unpredictability shift. Wouldn't a better answer be that the necessary assumption is that "no solidified lava has changed the direction of its magnetization unpredictably"?
 Steve Stein
PowerScore Staff
  • PowerScore Staff
  • Posts: 1153
  • Joined: Apr 11, 2011
|
#4580
Thanks for your response--you've touched upon a tricky concept:

In normal conversation, I agree, "not all" does seem to imply "some."

In the context of the LSAT, though, "not all" needs to be defined a bit more precisely: If there were 100 people on the guest list, and I tell you that "not all" of them attended the party, the only thing you know is that the number of guests was 99 or below--this even includes the possibility that none of the invitees showed up!

So, if "not all" leaves open the possibilities from zero to 99, the negated version becomes more clear:

"All solidified lava..."

I'd also suggest reviewing Lesson Two, which begins with a helpful discussion of logical opposition.

Good question! Let me know whether that clears this one up--thanks!

~Steve
 moshei24
  • Posts: 465
  • Joined: Mar 20, 2012
|
#4583
Steve,

I understood that some includes 99 or below. Not all logically includes anything but all. But that still doesn't answer my question. Yes, the negation would seem to make this question crystal clear that that is the best answer, but it still seems flawed. If we assume that not all the lava changes at random, that means that there is a possibility that anywhere between 0 (inclusive) and all can change at random. So if a random amount changes at random, there would still seem to be an issue with the argument. If there's a possibility that 99% changes at random, then there is a logical issue there. Just because that assumption negates the possibility that all of the lava changes at random doesn't really complete the argument.

I still feel that a more logical assumption answer would be that "none" of the lava changes at random. If "none" changes, then there is no possible issue.
 Steve Stein
PowerScore Staff
  • PowerScore Staff
  • Posts: 1153
  • Joined: Apr 11, 2011
|
#4586
Thanks for your response.

Remember that assumption negation is a two-step process. First, we negate the answer choice, finding its logical opposite, and then we see whether that negated version hurts the author's argument.

The correct answer choice on that one provides that "not all solidified lava..."
As you say, this includes every possibility but "all" (including "none").

So, what is the logical opposite of "not all"? All.

So, the logically negated version of "Not all solidified lava.."?

All solidified lava changed unpredictably. If that's the case, that weakens the author's argument.

Are you taking the class currently, and did you get a chance to take a look at the relevant portion of Lesson Two that deals with logical opposition? Please let me know if you do, and if anything is unclear.
 moshei24
  • Posts: 465
  • Joined: Mar 20, 2012
|
#4589
I took the course in the Spring.

I understand that the logical opposite weakens the argument. That has been clear from the start. But if the assumption was that no lava changes direction at random, the logical opposite being that "some" lava changes direction at random would also weaken the argument. Just because a logical opposite weakens the argument doesn't mean that it's the assumption. That's usually the case, but not always. That assumption itself would weaken the argument - without even negating it. If "not all" the lava changes direction at random, that assumption without negating it weakens the argument.

If there's a possibility that "some" lava changes direction at random, it could mean that the cause of "most" of the lava changing direction was randomness as opposed to the Earth changing over time. In order for the assumption to be logical, it would need to be that "none" of the lava changes at random, and at that point it becomes clear that it changes because the Earth changes. Do you see what I'm saying?
 moshei24
  • Posts: 465
  • Joined: Mar 20, 2012
|
#4597
I have not looked at lesson 2. The issue isn't with my lack of understanding the concepts; I believe the issue is that you aren't understanding the point I keep making. You keep answering my question the way you see it, not the way I'm saying it.

"The correct assumption was NOT that "no lava changes direction at random."
***I'm saying that that SHOULD be the correct assumption***

The correct assumption was that "not all lava changes direction at random".

These are very different.

If we were to logically negate the first, incorrectly written assumption, we get:

Some lava changes direction at random.***I said this WOULD weaken the author's argument. Some can mean 99%, which would weaken the argument.***

When we logically negate the actual answer choice we get:
ALL lava changes direction at random. This does weaken the author's argument."
***That also weaken the argument, but so does the assumption in the answer. 'some lava changing direction' also weakens the argument. The only thing that wouldn't weaken the argument is 'no lava changing direction.'

Do you see what I'm saying now?
 Steve Stein
PowerScore Staff
  • PowerScore Staff
  • Posts: 1153
  • Joined: Apr 11, 2011
|
#4598
I have not looked at lesson 2. The issue isn't with my lack of understanding the concepts; I believe the issue is that you aren't understanding the point I keep making.


Thanks for your response--sorry if I'm not being clear! Let's go back to the beginning and see if we can't clear this up. In two earlier posts, you said the following: "not all, which means some..."

Let's start with that statement. Do you recognize that this is not the case?

Please let me know--thanks!

~Steve

Get the most out of your LSAT Prep Plus subscription.

Analyze and track your performance with our Testing and Analytics Package.